Difference between revisions of "2016 AMC 12A Problems/Problem 3"

(Solution)
(Solution)
Line 1: Line 1:
  
 +
 +
==Problem==
 +
The remainder function can be defined for all real numbers <math>x</math> and <math>y</math> with <math>y\ne 0</math> by
 +
<math>\text{rem}(x,y)=x-y\Big\lfloor\frac{x}{y}\Big\rfloor}</math>,
 +
where <math>\Big\lfloor\frac{x}{y}\Big\rfloor</math> denotes the greatest integer less than or equal to <math>\frac{x}{y}</math>. What is the value of <math>\text{rem}\left(\frac{3}{8},-\frac{2}{5}\right)</math>?
 +
 +
<math>\textbf{(A)}\ -\frac{3}{8}\qquad\textbf{(B)}\ -\frac{1}{40}\qquad\textbf{(C)}\ 0\qquad\textbf{(D)}\ \frac{3}{8}\qquad\textbf{(E)}\ \frac{31}{40}</math>
  
 
==Solution==
 
==Solution==

Revision as of 23:43, 3 February 2016


Problem

The remainder function can be defined for all real numbers $x$ and $y$ with $y\ne 0$ by $\text{rem}(x,y)=x-y\Big\lfloor\frac{x}{y}\Big\rfloor}$ (Error compiling LaTeX. Unknown error_msg), where $\Big\lfloor\frac{x}{y}\Big\rfloor$ denotes the greatest integer less than or equal to $\frac{x}{y}$. What is the value of $\text{rem}\left(\frac{3}{8},-\frac{2}{5}\right)$?

$\textbf{(A)}\ -\frac{3}{8}\qquad\textbf{(B)}\ -\frac{1}{40}\qquad\textbf{(C)}\ 0\qquad\textbf{(D)}\ \frac{3}{8}\qquad\textbf{(E)}\ \frac{31}{40}$

Solution

\[\text{rem}\left(\frac{3}{8},-\frac{2}{5}\right)\] \[=\frac{3}{8}-\left(-\frac{2}{5}\right)\Big\lfloor\frac{\frac{3}{8}}{-\frac{2}{5}}\Big\rfloor\] \[=\frac{3}{8}+\left(\frac{2}{5}\right)\Big\lfloor -\frac{15}{16}\Big\rfloor\] \[=\frac{3}{8}+\left(\frac{2}{5}\right)\left(-1\right)\] \[=\frac{3}{8}-\frac{2}{5}\] \[=\boxed{\textbf{(B)}-\frac{1}{40}}\]